Welcome Guest!

  • IELTS Listening
  • IELTS Reading
  • IELTS Writing
  • IELTS Writing Task 1
  • IELTS Writing Task 2
  • IELTS Speaking
  • IELTS Speaking Part 1
  • IELTS Speaking Part 2
  • IELTS Speaking Part 3
  • IELTS Practice Tests
  • IELTS Listening Practice Tests
  • IELTS Reading Practice Tests
  • IELTS Writing Practice Tests
  • IELTS Speaking Practice Tests
  • All Courses
  • IELTS Online Classes
  • OET Online Classes
  • PTE Online Classes
  • CELPIP Online Classes
  • Free Live Classes
  • Australia PR
  • Germany Job Seeker Visa
  • Austria Job Seeker Visa
  • Sweden Job Seeker Visa
  • Study Abroad
  • Student Testimonials
  • Our Trainers
  • IELTS Webinar
  • Immigration Webinar

ielts-material

IELTS Direct Question Essays – Structure, Questions, Samples & Tips

Ann Smith

Updated On Nov 28, 2023

direct question essay example

Share on Whatsapp

Share on Email

Share on Linkedin

IELTS Direct Question Essays – Structure, Questions, Samples & Tips

Limited-Time Offer : Access a FREE 10-Day IELTS Study Plan!

The IELTS Writing Task 2, has an essay type known as the “Direct Question Essay,” which is a crucial component of the IELTS exam. It evaluates your ability to express your ideas, analyze a given topic, and provide a clear, well-structured response.

In this comprehensive guide, we will explore the structure of  IELTS Writing Task 2 : Direct Question Essays, delve into essential pointers, provide ten valuable tips to ace this task and furnish you with five sample questions and answers to boost your preparation.

Structure of a Direct Question Essay

A Direct Question Essay typically follows a specific format:

I agree that schools should place a greater emphasis on practical life skills. While traditional subjects are essential, practical skills like cooking, budgeting, and home repair equip students for real-life challenges. For instance, teaching basic culinary skills can promote healthier eating habits, reducing the prevalence of diet-related health issues.

Furthermore, budgeting knowledge empowers individuals to manage their finances effectively, reducing the risk of falling into debt. The ability to handle basic home repairs fosters independence and self-reliance, which are valuable qualities in adulthood.

By incorporating practical skills into the curriculum, schools prepare students for the responsibilities they will face in their daily lives. While academic knowledge is important, it is equally crucial to equip individuals with the tools they need to thrive in the real world. Thus, I firmly believe that schools should allocate more resources to teach practical life skills alongside traditional subjects, ensuring a holistic education that sets students on the path to success.

Banning smoking in public places offers numerous advantages. Firstly, it significantly improves public health by reducing exposure to harmful secondhand smoke. Non-smokers, including children and those with respiratory conditions, benefit from cleaner air, leading to a lower incidence of smoking-related health issues.

Secondly, such bans encourage individuals to quit smoking or reduce their consumption. When smoking is restricted in public spaces, smokers may be more inclined to quit as the inconvenience of finding a designated smoking area becomes apparent. This, in turn, decreases the overall demand for tobacco products and contributes to public health.

However, banning smoking in public places can lead to potential disadvantages. Businesses reliant on tobacco sales may experience a decline in revenue, potentially leading to layoffs or closures. Additionally, some smokers may resist the bans, leading to enforcement challenges and potential conflicts in public spaces.

In conclusion, the advantages of banning smoking in public places, such as improved public health and reduced smoking rates, outweigh the disadvantages. While businesses may face challenges, the long-term benefits to society as a whole are substantial.

Key Pointers for Direct Question Essays

  • Understand the Question:  Read the question carefully to grasp its requirements and focus. Identify keywords that dictate the scope of your response.
  • Plan Your Essay:  Spend a few minutes brainstorming and outlining your essay. Organize your thoughts and main ideas. A well-structured essay flows more naturally.
  • Clarity and Coherence:  Use clear and concise language. Ensure that your essay is easy to follow and logically organized. Employ appropriate transitions between sentences and paragraphs.
  • Stay on Topic:  Avoid straying from the main question. Irrelevant information can reduce the clarity of your essay and affect your score.
  • Word Count:  Adhere to the word count requirement. Going over or under the limit can result in point deductions.
  • Vocabulary and Grammar:  Showcase a wide vocabulary range and accurate grammar. Simple and complex sentences should be used appropriately to enhance the quality of your writing.
  • Examples and Details:  Support your ideas with relevant examples, facts, and details. This strengthens your arguments and demonstrates a deep understanding of the topic.
  • Counter Arguments:  Address counterarguments when relevant. This shows your ability to consider different perspectives and strengthens your position.
  • Time Management:  Allocate time wisely. Ensure you have enough time to review and edit your essay before submission.
  • Revision:  Always review and edit your essay. Look for errors in spelling, grammar, and clarity. A well-edited essay leaves a positive impression.

You can practice more Direct Question Essays  here .

Direct Question Essay: A Skill to Learn!

In conclusion, mastering the art of writing Direct Question Essays is achievable with practice, a clear understanding of the format, and attention to key pointers. By following the structure, incorporating the tips provided, and analyzing the sample questions and answers, you can significantly enhance your performance in the IELTS Writing Task 2. So, start practicing and make your writing shine on test day. Good luck!

Here are the 10 examples for the Direct question essay

Also, check :

  • Tips to Improve IELTS Writing Skills
  • IELTS Writing recent actual test
  • IELTS Band 9 essays
  • Advantage and Disadvantage Essays
  • IELTS Writing Task 1 Connectors

Frequently Asked Questions

How will you write an introduction in the IELTS Direct Question Essay?

What are the common errors the examinees make in the IELTS Direct Question Essay?

How can you score high marks for task achievement in the IELTS Direct Question Essay?

Can I use only one body paragraph in my IELTS Direct Question Essay?

Are IELTS Direct Question Essay and IELTS Double Question Essay the same?

Practice IELTS Writing Task 2 based on Essay types

ielts img

Start Preparing for IELTS: Get Your 10-Day Study Plan Today!

Ann Smith

Explore other Direct Question Essays

Some People Say the Main Way to Be Happy in Life is to Have a Lot of Money – IELTS Writing Task 2

Raajdeep Saha

Is freedom of speech necessary in a free society? – IELTS Writing Task 2

Kasturika Samanta

Many People Like to Wear Fashionable Clothes – IELTS Writing Task 2

Janice Thompson

Consumers Are Faced With Increasing Numbers of Advertisements From Competing Companies – IELTS Writing Task 2

Post your Comments

direct question essay example

Posted on Apr 15, 2022

Thank you very much for sharing this useful post.I’m sure this educational programs & sectors will be helpful for those who want to improve their skills in learning better English. So I expect you going on this.

Recent Articles

Some People Prefer to Eat at Restaurants While Others Prefer to Prepare and Eat at Home – IELTS Writing Task 2

Nehasri Ravishenbagam

Many People Complain that They Have Difficulties Getting Enough Sleep – IELTS Writing Task 2

Our Offices

Gurgaon city scape, gurgaon bptp.

Step 1 of 3

Great going .

Get a free session from trainer

Have you taken test before?

Please select any option

Get free eBook to excel in test

Please enter Email ID

Get support from an Band 9 trainer

Please enter phone number

Already Registered?

Select a date

Please select a date

Select a time (IST Time Zone)

Please select a time

Mark Your Calendar: Free Session with Expert on

Which exam are you preparing?

Great Going!

IELTS Charlie

Your Guide to IELTS Band 7

IELTS Direct Question Essay: tips, common mistakes, questions & essays

In this lesson we are going to look at how to answer an IELTS Direct Question essay.

You will learn about this  IELTS Writing Task 2  essay, using  authentic IELTS essay questions , plus the most common mistakes. And I will finish with an  IELTS model essay  written by me in response to a  sample IELTS essay question . So let’s get started!

What Is Your Task?

In this IELTS question type, you are usually asked 2 questions. Your task is to simply answer these questions!

Often, one or both of the 2 questions come from one of the other 4 question types, so you might be asked to say whether you agree or disagree, or to discuss the disadvantages of something, or to suggest some solutions.

So it’s really important to read the question carefully!

Here is an example direct questions essay task:

Some people spend most of their lives living close to where they were born. 

What might be the reasons for this? 

What are the advantages and disadvantages? 

Cambridge IELTS 16 General Training Test 3

As you can see, this task has 2 questions. The 2nd question is about advantages and disadvantages, so the task takes the 2nd question from one of the other essay question types.

How To Plan An IELTS Direct Question Essay

If you are aiming for a high band score (band 7 and above) it is absolutely vital that you plan your essay. A good plan will help you to see if you have answered the question, developed your ideas and organised them BEFORE you start writing.

We’re going to plan an essay using my  4 Step Planning Process .

4 Step Planning Process

Step 1: Understand The Task

First, you need to make sure you understand exactly what you need to write about. So you need to read the question carefully, not quickly!

Think about these three questions:

What is the topic about?

What is the topic NOT about?

How should you respond to the topic?

Let’s go back to this essay question, and answer those 3 questions:

  • The topic is about people who live near to the village / town of their birth for most of their lives.
  • (The word “birthplace” implies the village /  town of their birth, NOT the country of their birth.)
  • It’s not about people who live ALL their lives near their birthplace.
  • Because “birthplace” implies a village / town / city, it’s not about people who rarely go outside their country.
  • The two questions, “what might be the reasons for this?” and “what are the advantages and disadvantages” tell you how to respond to the topic. So make sure you answer these questions in your plan.
  • Many students might forget to answer the first question, and just write about the advantages and disadvantages. This will limit your band score for Task Response to Band 5.

Step 2: Decide Your Position

Next, you need to decide your position. In other words, you need to decide what you think.

In a direct questions essay, your position is your answers to the two questions .

So in our example above, your position is your answer to the 2 questions:

  • what might be the reasons for people spending most of their lives near their birthplace?
  • what are the advantages and disadvantages of spending most of your life near your birthplace?

Step 3: Extend Your Ideas

When you decided your position, you may have started thinking about the reasons for your position, the reasons for your answer. In other words, WHY are you taking this view?

Giving reasons for your view is essential in an IELTS essay. In fact, all IELTS questions tell you to “give reasons for your answer”. So in Step 3, you need to think about your reasons a little more.

However, just presenting your  reasons is not enough. You need to develop them.

The two best ways of developing your ideas is by:

  • giving explanations of what you mean
  • giving specific examples which illustrate what you mean

Together, these add more detail to your answer.

You MUST do this to get Band 7. If you fail to develop your ideas in detail, your band score for Task Response may be limited to Band 6.

Read more about how to develop your ideas in an IELTS essay.

Step 4: Structure Your Essay

The final step in the planning process is to structure your essay. This simply means deciding which main ideas to put in which paragraphs.

I would recommend a simple structure like this:

  • Paragraph 1: introduce the essay
  • Paragraph 2: discuss your answer to the 1st question
  • Paragraph 3: discuss your answer to the 2nd question
  • Paragraph 4: summarise your ideas.

ielts-direct-question-essay

How To Write Your IELTS Direct Question Essay

Let’s go through how to write the different parts of the essay.

How To Write The Introduction

In the introduction to an IELTS Direct Questions essay, you need to do two things:

  • briefly introduce the topic of the essay
  • briefly say what you are going to write about

Introduce The Topic

You should begin with a background sentence which introduces your reader to the topic of the essay. The best way to do this is to paraphrase the topic statement.

How To Paraphrase

Think about the meaning of the topic statement, and briefly rewrite it using your own words. Try not to use the same grammatical structures as in the essay question, and try to move language around. In other words, be flexible. This is important if you are aiming for a Band 7 or higher.

In the example essay question above, the topic statement said:

“Some people spend most of their lives living close to where they were born.”

Here is one way of paraphrasing this:

“Despite opportunities to travel widely in the modern world, many people still live most of their lives not far from their birthplace.”

This sentence has a similar meaning as the original sentence, but uses different vocabulary and different grammatical structures.

Say What You Are Going To Write About

In an IELTS Direct Questions essays, it’s a good idea to briefly say what you are going to write about – in other words, say that you are going to answer the two questions.

In our example essay above, we need to answer two questions:

  • What might be the reasons for this?
  • What are the advantages and disadvantages?

So I could write:

“This essay will consider the reasons for this tendency, along with the possible benefits and drawbacks.”

How To Write The Body Paragraphs

In an IELTS Direct Questions essay, you simply need to answer the questions in the body paragraphs.

Write the answer to each question in a separate paragraph.

Direct questions essays can vary quite a lot, so you need to be flexible in your paragraphing.

In our example essay, the two questions are:

So, the first body paragraphs could contain:

  • A reason why people live most of their lives near their birthplace
  • A more detailed explanation of this reason
  • An example which illustrates this reason
  • You can also include a 2nd reason in the same paragraph.

The second body paragraph could contain:

  • One advantage of living most of their life near their birthplace
  • A more detailed explanation of this advantage
  • An example which illustrates this advantage

You could include a disadvantage in the same paragraph, but I would personally write it in a separate paragraph:

  • One disadvantage of living most of their life near their birthplace
  • A more detailed explanation of this disadvantage
  • An example which illustrates this disadvantage

(You probably only have enough time to write one advantage and one disadvantage in this essay, because you also have to write about the reasons for living in one place.)

You can read more about  developing your ideas here .

How To Write The Conclusion

In the conclusion to an IELTS Direct Questions essay, you need to do one thing:

  • summarise your main points

Do NOT write any new ideas in your conclusion. If you think of new ideas while writing your conclusion, forget them! It’s too late.

Common Mistakes in an IELTS Direct Question Essay

These are the most common mistakes made by Test Takers when writing an IELTS Direct Questions essay:

  • not reading the question carefully enough. The questions in these essay types can vary a lot, so don’t read the questions quickly. Read them carefully.
  • Writing an overly general statement about the topic in the introduction (e.g. Education is a topic of hot debate.
  • Your main ideas are not explained and illustrated enough. You need to develop all of your ideas to get a band 7 and higher.
  • Using memorised phrases (e.g. “a hot topic”, “in a nutshell”, “pros and cons”)
  • Using “research studies” as examples: examples should illustrate your ideas, not prove them. Read about  how to use examples in IELTS essays .
  • Trying to use rare or “novel” language: examiners are looking for groups of words used naturally, not rare words.

Sample IELTS Direct Question Essay Questions

In some countries, more and more people are becoming interested in finding out about the history of the house or building they live in. What are the reasons for this? How can people research this?

(Cambridge IELTS 16 Academic Test 1)

In their advertising, businesses nowadays usually emphasise that their products are new in some way. Why is this? Do you think it is a positive or negative development?

(Cambridge IELTS 16 Academic Test 2)

In some countries, owning a home rather than renting one is very important for people. Why might this be the case? Do you think this is a positive or negative situation?

( Cambridge IELTS 15 Academic Test 1 )

In many countries today, crime novels and TV crime dramas are becoming more and more popular. Why do you think these books and TV shows are popular? What is your opinion of crime fiction and TV crime dramas?

(Cambridge IELTS 15 General Training Test 1)

Model IELTS Direct Question Essays

Here is an IELTS Direction Questions Essay that I wrote in response to this task:

In many countries today, crime novels and TV crime dramas are becoming more and more popular.

Why do you think these books and TV shows are popular?

What is your opinion of crime fiction and TV crime dramas?

Stories about criminal activity, both fictional and real-life, have become increasingly popular over the last few decades. There are many possible reasons for this, but the two primary ones that I can think of are the underlying desire of people to see good overcome evil, and a fascination with criminal lifestyles.

Almost all stories about crime, whether in print or on TV, are about good people, such as detectives and law-abiding civilians, triumphing over bad people, namely criminals. We often see this in fictional detective stories, where an otherwise ordinary person uses their intellect and skill to identify evil criminal masterminds. A good example of this is Miss Marple, an elderly woman who always manages to track down and apprehend evil criminals.

A second reason is that people have a fascination with the lives of criminals. Perhaps this is to do with people’s need for escapism. One of the most popular crime dramas in the UK of the last 20 years was ‘Legend’, a dramatisation of the lives of the Kray Twins, two violent London gang leaders of the 1960s. The film, which I watched on TV, portrayed their violent behaviour, along with their opulent and chaotic lifestyles, and I do feel that people find this compelling viewing, despite how it shows evil people succeeding.

Personally, unless it is related to real-life stories, I have little interest in either crime fiction or crime drama. I find their plots too repetitive. With true crime stories, however, I can learn something about social history and psychology. Why, for example, do people turn to lives of crime? Is it simply for money, or are they motivated by power as well? And what causes people to join gangs and follow people like the Krays? These are all interesting questions.

In summary, a desire to see good triumph over evil, along with a fascination with evil, are two reasons I think underlie the popularity of crime stories, but my interest in them is mainly limited to dramatisations of real lives.

(335 words)

Read my full plan and comments for this essay.

More IELTS Direct Question Sample Essays

Share this:.

  • Click to share on Facebook (Opens in new window)
  • Click to share on WhatsApp (Opens in new window)
  • Click to share on Twitter (Opens in new window)
  • Click to share on LinkedIn (Opens in new window)
  • Click to share on Reddit (Opens in new window)
  • Click to share on Pinterest (Opens in new window)
  • Click to share on Telegram (Opens in new window)
  • Click to share on Pocket (Opens in new window)
  • Click to print (Opens in new window)
  • Click to share on Tumblr (Opens in new window)

Related Posts

direct question essay example

About the author

Charlie is a former IELTS Examiner with 25 years' teaching experience all over the world. His courses, for both English language learners and teachers, have been taken by over 100,000 students in over 160 countries around the world.

IELTS® is a registered trademark of Cambridge English Language Assessment, the British Council, and IDP Education Australia. By using this website, you agree that you fully understand that ieltscharlie.com is not affiliated, approved or endorsed by Cambridge English Language Assessment, the British Council, or IDP Education Australia.

Unit 22489, PO Box 6945, London, W1A 6US, United Kingdom

© IELTSCharlie

Privacy Overview

Discover the 7 STEPS to BAND 7 in IELTS Writing Task 2

  • Phone: +91 8466016171
  • Whatsapp: +91 8208375580
  • Email: contact@leapscholar.com

Direct Question Essay IELTS: Tips, Structure, Sample Questions & Answers

  • Updated On December 12, 2023
  • Published In IELTS Preparation 💻

Over the years, the IELTS writing task 2 has included a variety of essay types such as opinion essays, problem/solution, or advantage/disadvantage essays. Direct Question Essay IELTS is also known as a “double question” or “two questions” essay as it is composed of two distinct questions.

Table of Contents

The direct questions are very specific and direct as the name suggests. Though direct questions are not common in IELTS writing tasks, if you master this type of essay, it will be easier for you to structure all the other types of essays as well. The direct questions in IELTS have to be answered in 250 words. The same will be scored based on the essay’s vocabulary, grammar, and word count.

Direct Question Essay IELTS

Leap Advantage Virtual Spot Offer Event on May 10th 2024

study in canada

Last call to secure your spot for Masters in STEM Program in the US for Fall ’24 exclusively for graduates and above.

LA Banner Web

Tips to Ace the Direct Question Essay IELTS Writing Task 2

Though there are several relevant examples and solutions essays for the direct question task available over the internet, some of the tips that can be practised for writing a Direct Question Essay in the IELTS test are:

  • Take some time to understand the question type and then plan the answer before you actually start attempting the IELTS writing task.
  • Write the essay in a formal tone.
  • Strictly follow the word count. The essay should at least be 250 words, not less than that. You will get 40 minutes to write the exam, so aim for 250-270 words.
  • Follow a proper content structure, wherein you begin with an introduction paragraph and then end the essay with a crisp conclusion paragraph, with a two-part body paragraph between these two.
  • Make sure that your introduction is strong and in coherence with the assigned topic or asked question. Your introduction is the part that can get your reader hooked on your essay and encourage them to read further.
  • Answer the questions asked and state the reasons for your statements in the main body paragraph of the essay.
  • Use proper and refined grammar and vocabulary. It helps the examiner evaluate your knowledge of the language and proficiency in the same.
  • Provide relevant real-life examples in your content. Supporting your statements with specific reasons and proper examples that emphasise your point can help you get a better IELTS score.

IELTS Direct Question Essay Structure

You must follow a certain structure while writing your Direct Question Essay IELTS. The structure for the same is as follows:

Crack IELTS Exam in first attempt

Direct Question Essay IELTS: Tips, Structure, Sample Questions & Answers

Attend Leap’s free masterclass to get tips, tricks and advance strategies to crack IELTS exam in first attempt

Direct Question Essay IELTS: Tips, Structure, Sample Questions & Answers

Introduction Paragraph

Your essay should always begin with an introduction paragraph. You can paraphrase the question in this paragraph and also give an introduction to what you will be discussing further in the body paragraphs.

Body Paragraph 1

In this paragraph, you can address the question and discuss the response to the first question, along with relevant examples.

Body Paragraph 2

Herein, you can discuss the details and answer the second question or the second part of the question, along with examples.

Conclusion Paragraph

Conclude the entire essay here, and summarize the content and main points of the essay in the conclusion part.

Most of the questions in the IELTS exams come with specific prompts that are easy to recognise and help you to shape your answers further. Some of these prompts are:

  • Is this development positive or negative?
  • Discuss the views on X and give your opinion on the same.
  • Do the advantages of X outweigh its disadvantages?
  • To what extent do you agree or disagree with the given statement.

Also Read: Updated List Vocabuly for IELTS

Direct Essay Question: Evaluation Factors

Some of the factors that are used for marking and evaluating Direct Question Essay IELTS – Writing Task 2 are:

Task Achievement 

The essay should provide a complete answer to both the questions asked and support the same with relevant and real-life examples.

Lexical Resource

The essay should be written with a wide range of vocabulary, and there should be no errors in the essay.

Grammatical Range and Accuracy

The essay should not have any grammatical errors. The sentences should have a different range of structures. 

Coherence & Cohesion

The solution essay should be divided into clear paragraphs that must maintain a logical flow throughout the whole essay. Each body paragraph should highlight one main idea. The main body paragraphs should have cohesive links to ensure a proper flow in the essay.

Direct Essay IELTS Sample Questions

Practice is the only way to improve your writing skills and ace your IELTS writing tasks. Some of the topics and related sample questions that you can study for your Direct Question Essay IELTS practice are:

News plays a very important role in the lives of most people. Why is news so important to many people? Why is a lot of news dedicated to the bad news sections? Should news channels give more news and focus on the good news instead?

What do you understand by ‘happiness’? How can people achieve it? Can money or material belongings help a person become happy?

In some parts of the world, people are becoming more focused on connecting with older people and researching their history. Why do you think people want to do this? Is it a positive or negative development?

There is too much news and information circulating on the internet nowadays. Is the information on the internet reliable? How can it be controlled?

How can success be measured? How can one become a successful person? Do you think wealth is an important factor or the best measure of success for a person?

Companies often employ a variety of methods to boost and improve the sales of their products. What are the generally employed methods? Out of them, which one is the most effective method?

It is often said, ‘when in Rome, do as the Romans do’. Do you agree with this? Do you think that people should adapt to the culture of the country that they visit or live in?

Also Read: IELTS Band Score 2022:Chart & Scoring System

Foreign Language Direct Question Essay IELTS Sample 

A sample of the Direct Question Essay IELTS Writing Test 2 is given below for your reference:

In advertising nowadays, businesses usually highlight and emphasise that their products are new in some way. Why do businesses practice this? Do you think this development is a positive practice or a negative one?

Solution Essay

Businesses have recently been highlighting through their promotional materials that their products include ‘new features’ different from those that are already existing. There can be many reasons for incorporating this move, such as gaining other people’s attention or justifying the increase in the price of their products. Though this is a positive strategy, it also has some negative consequences.

To begin with, businesses always highlight the latest design features or technologies of their products to attract a lot of their customers. This, in my opinion, is a good step ahead because it will make the decision-making process for the customers more manageable. Customers would not have to conduct in-depth research on the products as the marketing team would have already done the basic groundwork for them. Businesses can also justify the price increase and therefore gain more profit.

On the other hand, this marketing strategy has some negative consequences. Businesses can make minor product changes and claim that they have changed them. In this way, they can manipulate their targeted customers by informing them that their products are equipped with the most recent features compared to previous or existing versions. They can also raise the price of the products while making only minor changes. As a result, in some ways, this marketing strategy can be deceptive to consumers.

Lastly, in their promotional activities, businesses place a greater emphasis on the new features of their products. This marketing strategy has resulted in some positive developments, such as targeted customers purchasing their products. However, by manipulating capitalists, this marketing method can have a negative impact on customers.

Direct Question Essay IELTS

Direct Questions are the specific two-part questions asked in IELTS Writing Exam Part 2. 40 minutes are provided to write an essay of 250 words for Direct Question IELTS. 

You can practice for the Direct Question Essay IELTS by writing on various topics available, including the internet itself, lifestyle, success, news, happiness, industry trends, and the like. It is very important to understand the question and its parts before attempting the direct question essay. You must form an idea with a precise opinion and relevant examples. The word limit is a factor that you should strictly abide by. Try to frame the essay logically so that there is a consistent flow throughout all your arguments. 

As they say, ‘Practise makes perfect,’ so keep practising to achieve your goal band score. 

For more related content, visit LeapScholar .

Related Articles: 

  • IELTS Social Media Essays
  • IELTS Essay Topic on Ideal Society

Frequently Asked Questions

1. how long is the ielts writing test.

Ans. The IELTS Writing Test is of a total duration of 1 hour. Of this, 20 minutes should be spent on the first writing task, and 40 minutes should be dedicated to writing task 2.

2. What are the most common errors candidates make in the Direct Essay Questions IELTS?

Ans. Many candidates attempting the IELTS make common errors such as not answering both parts of the questions completely, answering just one question, not understanding the question type properly, and not emphasizing both the answers in the introduction paragraph. These errors can lead to an unbalanced essay that will affect your IELTS task achievement score.

3. How can Direct Essay Questions be distinguished from other types of questions? 

Ans. IELTS Direct Question Essays are also referred to as IELTS Double Question Essays. Two main characteristics that can be used to distinguish these questions from others are that they have one statement and two different questions given under it, and the questions given may or may not be linked to each other. Therefore, the candidate should spend some time analysing the question, developing the ideas, and planning their answer to write.

Know more about IELTS

Get free speaking practice samples.

  • Describe a Skill That you can teach other people
  • Describe a Place you Visited Where the Air was Polluted
  • Describe a Famous Person that you are Interested in
  • Describe a Course That You Want to Learn
  • Describe a Person who Solved a Problem in Smart Way
  • Describe a Prize That You Received
  • Describe a Volunteering Experience You Have Had
  • Describe a Piece of Good News
  • Describe Something you Taught to Your Friend
  • Talk About an Interesting Old Person you Met Recently
  • Describe a Dinner you Really Enjoyed
  • Describe a Story or Novel That Was Interesting to You
  • Describe a Time When you Shared Something with Others
  • Describe a Toy you Liked in Childhood
  • Describe an Interesting Neighbour
  • Describe a Competition You Would Like to Take Part In
  • Describe an interesting activity that you remember enjoying most in your Primary School
  • Describe Your Childhood Friend
  • Describe an Object You Find Particularly Beautiful
  • Describe a Place Where you are Able to Relax
  • Describe a person you know who likes to talk a lot
  • Describe a place where you would like to go to relax
  • Describe a period when you were busy
  • Describe a long-term goal you would like to achieve
  • Describe a situation when you helped someone
  • Describe a job you would not like to do in the future
  • Describe a time when you used a map
  • Describe a skill that you learned in your childhood
  • Describe an unusual holiday you had
  • Describe an exciting activity that you experienced with someone else
  • Describe a person who is good at making people feel welcome in his/her home
  • Describe A Time You Successfully Did Something Difficult
  • Describe Something in Your Country That You Are Interested In
  • Describe a Beautiful City
  • Describe something you do regularly that helps you work and study better
  • Describe a photo that you took and are proud of
  • Describe a party that you enjoyed
  • Describe a useful object in your home that you can’t live without
  • Describe a good advertisement that you think is useful
  • Describe an occasion when someone or something made noise
  • Describe a sportsperson that you admire
  • Describe something that you picked up that was thrown by someone else
  • Describe a time that something changed your life in good ways
  • Describe a successful person who you once studied or worked with

Get Free Reading Practice Samples

  • William henry perkin reading answers
  • Why zoos are good reading answers
  • Bioluminescence reading answers
  • Classifying societies reading answers
  • Artificial artists reading answers
  • Jargon reading answers
  • The Origins of Laughter Reading Answers
  • The Innovation of Grocery Stores Reading Answers
  • All About the Otter Reading Answers
  • The Triune Brain Reading Answers
  • Saving the Soil Reading Answers
  • Trans Fatty Acids Reading Answers
  • A Disaster of Titanic Proportions Reading Answers
  • Why Companies Should Welcome Disorder Reading Answers
  • Book Review Reading Answers
  • Tea Times Reading Answers
  • Why are Finland's Schools Successful Reading Answers
  • Intelligence and Giftedness Reading Answers
  • Animal Minds Parrot Alex Reading Answers
  • Crop Growing Skyscrapers Reading Answers
  • Secrets of the Swarm Reading Answers
  • Walking with Dinosaurs Reading Answers
  • The Development of Travel Under the Ocean Reading Answers
  • What's so Funny Reading Answers
  • The Culture of Chimpanzee Reading Answers
  • Clutter Bugs Beware Reading Answer
  • Stepwells Reading Answers
  • Glaciers Reading Answers

Get Free Writing Practice Samples

  • Advantages and Disadvantages Essay with Sample Answers
  • Agree and Disagree Essay with Sample Answers
  • Problem Solution Essay Topic with Sample Answers
  • Every year several languages die out
  • Positive or Negative Development Essay with Sample Answers
  • Honesty is the best policy essay
  • Online shopping essay
  • Environment essay topics
  • Prevention is better than cure essay

Get Free Listening Practice Samples

  • Family excursions listening answers
  • Public library listening answers
  • Hiring a public room listening answers
  • Notes on social programme listening answers
  • Accommodation request form listening answers
  • Transport survey listening answers
  • Mic house agency repairs listening answers
  • Holiday rentals listening answer
  • Job enquiry listening answers
  • Homestay application listening answers
  • Library information listening answers
  • Free activities in the burnham area listening answers

IELTS Important Information

  • IELTS Exam Date
  • IELTS Exam Fee
  • IELTS Modules
  • IELTS Speaking Practice Test
  • IELTS Writing Practice Test
  • IELTS Reading Practice Test
  • IELTS Listening Practice Test
  • IELTS Test Centres
  • IELTS Results
  • Types of IELTS
  • IELTS Pattern
  • IELTS Exam Eligibilty
  • IELTS Slot Booking
  • IELTS Band Score
  • IELTS Registration
  • IELTS Books
  • IELTS Preparation
  • IELTS Practice Test
  • IELTS Speaking Cue Card
  • IELTS Speaking Part 1
  • IELTS Writing Task 1
  • IELTS Writing Task 2
  • Task 1 Pie Chart
  • Task 1 Table Chart
  • Task 1 Bar Graph
  • Task 1 Line Graph
  • Task 1 Diagram
  • IELTS Computer Based Test
  • IELTS Paper Based Test
  • IELTS One Skill Retake
  • IELTS for UKVI
  • IELTS Vocabulary

IELTS Test Centre and Dates in India

  • IELTS Test Centre and Dates in Hyderabad
  • IELTS Test Centre and Dates in Bangalore
  • IELTS Test Centre and Dates in Chennai
  • IELTS Test Centre and Dates in Amritsar
  • IELTS Centre and Dates in Ludhiana
  • IELTS Test Centre and Dates in Mumbai
  • IELTS Test Centres and Dates in Ahmedabad
  • IELTS Centre and Dates in Delhi
  • IELTS Test Centres and Dates in Chandigarh
  • IELTS Center and Dates in Pune

Avatar photo

Kanika Singh

Kanika took a detour from academics for the corporate world. Now she works as a senior editor specialising in content creation, marketing, and strategy.

Related Posts

common words used in the daily life

A Guide on Most Common English Words Used in Daily Life

ielts score

Top IELTS Tips and Tricks For Best IELTS Scores 2024

Problem Solution Essay IELTS

60+ IELTS Essay Topics 2024: IELTS Writing Task 2 Samples & Tips

Trending now.

early childhood

  • Ebooks & Courses
  • Practice Tests

How to Plan & Write IELTS Double Question Essays

IELTS double question essays are also known as ‘direct question’ or ‘two questions’ essays. They are distinguished by two characteristics:

  • They have one statement with two different questions after it.
  • The questions may or may not be linked.

Here are 3 examples:

1) Fossil fuels are essential for producing electricity, powering industry and fueling transportation. However, one day we will reach a point when all the world’s fossil fuels have been depleted.

How can we conserve these resources?

What are some alternatives to fossil fuels?

2) Some parents buy their children whatever they ask for, and allow their children to do whatever they want.

Is this a good way to raise children?

What consequences could this style of parenting have for children as they get older?

3) The arts, including art, music and theatre are considered to be important in society.

Do you think the arts still have a place amongst our modern lifestyles?

Should the arts be included in the school curriculum?

In this lesson, I’m going to demonstrate step-by-step how to plan and write IELTS double question essays.

Here’s what we’ll be covering:

  • 3 Common mistakes
  • Essay structure
  • How to plan
  • How to write an introduction
  • How to write main body paragraphs
  • How to write a conclusion

Want to watch and listen to this lesson?

Click on this video.

Click the links to see lessons on each of these Task 2 essay writing topics. 

Once you understand the process, practice on past questions. Take your time at first and gradually speed up until you can plan and write an essay of at least 250 words in the 40 minutes allowed in the exam.

3 Common Mistakes

These three errors are common in IELTS double question essays.

  • Not answering both questions fully.
  • Not outlining both answers in the introduction.
  • Mistaking it for one of the other essay types.

Many students make the mistake of only answering one of the questions, or focusing more on one question than the other which leads to an unbalanced essay. Both these errors will seriously affect your score for task achievement.

You must outline everything you are going to write about in the introduction. This is your blueprint for the whole essay. I’ll show you how to do this and get your essay off to a great start.

It’s easy to mistake IELTS double question essays for one of the other four types of Task 2 essays, especially opinion or discussion essays. Each should be answered in a slightly different way.

Analysing the question properly is essential to avoiding this error. I’ll also show you how to do this and give you a simple 4 part structure for planning your essay.

Essay Structure

Let’s look at this essay structure straight away. You can use it to write any IELTS double question essay. It’s easy to learn and will enable you to quickly plan and write a high-level essay.

1)  Introduction  

  •   Paraphrase the question 
  •   Outline sentence – state your answer to both questions

2)  Main body paragraph 1 – Answer question 1

  • Topic sentence – state your answer
  • Explanation – develop the idea
  • Example – give an example

3)  Main body paragraph 2 – Answer question 2

4)  Conclusion Summarise both questions and answers

This structure will give us a well-balanced essay with 4 paragraphs.

We now need some ideas to add to the structure and we’ll have everything we need for our essay.

How To Plan IELTS Double Question Essays

Here’s the question we’re going to be answering in our model essay followed by the 3 steps of the planning process.

Fossil fuels are essential for producing electricity, powering industry and fueling transportation. However, one day we will reach a point when all the world’s fossil fuels have been depleted.

  • Analyse the question
  • Generate ideas
  • Identify vocabulary

# 1  Analyse the question

This is an essential step in the planning process and will ensure that you answer the question fully. It’s quick and easy to do. You just need to identify 3 different types of words:

  • Topic words
  • Other keywords
  • Instruction words

Topics words  are the ones that identify the general subject of the question and will be found in the statement part of the question.

Fossil fuels  are essential for producing electricity, powering industry and fueling transportation. However, one day we will reach a point when all the world’s  fossil fuels  have been depleted.

So, this question is about ‘ fossil fuels ’.

Many people will do this first step of the process and then write about the topic in general. This is a serious mistake and leads to low marks for task achievement.

Now that we know what the general topic is, we need to understand exactly what aspect of fossil fuels we're being asked to write about.

The  other keywords  in the question tell you the specific things you must write about. For IELTS double question essays, these will often be in the instructions, that is, the actual questions.

How can we  conserve  these resources?

What are some  alternatives  to fossil fuels?

By highlighting these words, it’s easy to identify the topics. Your essay must only include ideas relevant to these ideas.

The  instruction words  are the questions themselves. These tell you exactly what type of information is required and each will become the topic for one of the two main body paragraphs.

The first body paragraph will answer the first question (How?) and the second body paragraph will answer the second question (What?).

# 2  Generate ideas

The next task is to generate some ideas to write about.

There are several different ways to think up ideas. I cover them fully on the  IELTS Essay Planning  page.

We’re going to use the ‘friends technique’. This is the method I prefer as it allows you to take a step back from the stress of the exam situation and think more calmly.

Here’s how it works. Imagine that you are in a casual conversation with a friend over a cup of coffee and they ask you this question. What are the first thoughts to come into your head? Plan your essay around these ideas.

Doing this will help you to come up with simple answers in everyday language rather than straining your brain to think of amazing ideas using high-level language, which isn’t necessary.

You might want to try this yourself before reading on for my ideas.

Here are my ideas as I thought of them:

How can we conserve these resources? 

  • Become more energy conscious & more energy efficient
  • Use more renewable energy sources – solar panels
  • All new homes should be built with solar panels on
  • Use car less – walk, cycle, public transport, only travel when really necessary
  • Energy-efficient light bulbs
  • Solar power
  • Wave energy
  • Tidal energy
  • Biomass energy
  • Geothermal energy

Don’t spend long on this as you only need one or two ideas.

There is so much to write about this topic that we have to be very careful we don’t try to include too many different ideas and just end up with a list for each question rather than a well-developed essay.

Choose one main idea for each part of the question. My advice on making your selection is to choose ideas that you can quickly think of examples for.

Here are my choices:

  • Use car less – walk, cycle, public transport
  • Natural forces – solar & wind power, wave & tidal energy

We’re almost ready to start writing our IELTS double question essay but first, we have one other small task to do.

# 3  Vocabulary

During the planning stage, quickly jot down some vocabulary that comes to mind as you decide which ideas you are going to write about, especially synonyms of key words. This will save you having to stop and think of the right language while you’re writing.

For the ideas I’ve chosen, useful words will include:

  • sustainable  
  • renewable energy
  • energy-efficient

With that done, we can focus on the first paragraph of the essay – the introduction.

How To Write an Introduction

The best introductions to IELTS double question essays have a simple 2 part structure:

1)   Paraphrase the question

2)   Outline sentence – state your answer to both questions

  • Have 2-3 sentences
  • Be 40-60 words long
  • Take 5 minutes to write

1)  Paraphrase the question

Start your introduction by paraphrasing the statement part of the question.

Question statement:

Paraphrased question:  

The world is currently reliant on oil, coal and natural gas for the majority of its energy requirements but there will come a time when these run out.

We are simply saying the same thing in a different way and using different vocabulary.

2)  Outline statement

Now we need to add an  outline statement  where we outline the two main points that we’ll cover in the rest of the essay, that is, the answers to the two questions.

We need to be very specific about what we are going to write about.

Here's a reminder of the ideas I’ve chosen to answer the two questions:

  • Natural forces –solar & wind power, wave & tidal energy

Outl ine statement:  

This essay will discuss how we can help to prevent our non-renewable resources from becoming depleted by using our cars less frequently and it will name some natural forces that can be harnessed to generate power.

Note my use of synonyms to replace key words in the question. You don’t have to replace every key word but do so where possible whilst ensuring that your language sounds natural.

So, let’s bring the two elements of our introduction together.

     Introduction

direct question essay example

This introduction achieves three important functions:

  • It shows the examiner that you understand the question.
  • It acts as a guide to the examiner as to what your essay is about.
  • It also helps to keep you focused and on track as you write.

The two ideas in your introduction will become your two main body paragraphs.

Main body paragraph 1  –  Use car less – walk, cycle, public transport

Main body paragraph 2  –  Renewable energy / natural forces – solar & wind power, wave & tidal energy

How To Write Main Body Paragraphs

Main body paragraphs in IELTS double question essays should contain 3 things:

  • Explanation –  develop the idea

Main Body Paragraph 1  – Answer question 1

The  topic sentence  summarises the main idea of the paragraph. That’s all it needs to do so it doesn’t have to be complicated.

It plays an important role in ensuring that your ideas flow logically from one to another. It does this by acting as a signpost for what is to come next, that is, what the paragraph will be about.

If you maintain a clear development of ideas throughout your essay, you will get high marks for task achievement and cohesion and coherence.

We’ll now take the idea for our first main body paragraph and create our topic sentence.

Main body paragraph 1  –  Use car less – walk, cycle, public transport, only travel when really necessary

Topic sentence:  

Conserving energy is a responsibility of every individual and an important way in which we can all do our bit is to use more energy-efficient means of transport. 

Next, we must write an  explanation sentence  that develops the idea.

Explanation sentence: 

The easiest way to do this is to leave the car at home and walk or cycle to our destination if it isn’t too far away, or take public transport for longer journeys. Another way to reduce our fuel consumption is to car share.

Finally, we add an  example  to support our main point. If you can’t think of a real example, it’s fine to make one up, as long as it’s believable. The examiner isn’t going to check your facts. Alternative, you could add another piece of information to support your idea but an example is better.

Example sentence:

Whenever my friends and I get together for coffee, we agree to meet up at a café that we can each get to without having to drive our cars there. We usually go on foot or ride our bikes. If everyone made small decisions like this, it would make a real difference.

That’s the 3 parts of our first main body paragraph complete. Here’s the finished paragraph.

direct question essay example

We now follow the same process for our second main body paragraph.

Main Body Paragraph 2  – Answer question 2

Again, we’ll now take the idea I’ve chosen for this paragraph and create our topic sentence.

Main body paragraph 2  –  Renewable energy / natural forces –   solar & wind power, wave & tidal energy

Topic sentence:

The most sustainable alternatives to fossil fuels are the generation of power from natural forces such as the sun, wind and oceans.

Now for the  explanation  where we expand on this idea.

Explanation sentence:

S olar and wind power are already widely used across the world but it is wave power and tidal energy that have the greatest untapped potential to provide for our energy needs in the future.

Finally, an  example  to support our main point.

A report recently commissioned in the United Kingdom estimates that tidal energy could meet as much as  20% of the UK’s current electricity demands once the technology being developed is operational. Wave energy converters are expected to prove equally successful in the long-term.

That’s the 3 parts of our second main body paragraph complete. Here’s the finished paragraph.

direct question essay example

Now we need a conclusion and our IELTS double question essay is done.

How To Write a Conclusion

The conclusion is a summary of the main points in your essay and can often be done in a single sentence. It should never introduce new ideas.

If you're below the minimum 250 words after you’ve written your conclusion, you can add a prediction or recommendation statement.

Our essay is already over the minimum word limit so we don’t need this extra sentence but you can learn more about how to write a prediction or recommendation statement for IELTS double question essays on the  Task 2 Conclusions  page.

The conclusion is the easiest sentence in the essay to write but one of the most important.

A good conclusion to an IELTS double question essay will:

  • Neatly end the essay
  • Link all your ideas together
  • Sum up your argument or opinion
  • Answer the question

If you achieve this, you’ll improve your score for both task achievement and cohesion and coherence which together make up 50% of the overall marks. Without a conclusion, you’ll score below band 6 for task achievement.

You can start almost any final paragraph of an IELTS double question essay with the words:

  • In conclusion

        or

  • To conclude

Now all you need to do is briefly summarise the main ideas into one or two sentences.

Here’s a top tip . Go back and read the introduction to the essay because this is also a summary of the essay. It outlines what you are going to write about.

To create a great conclusion, you simply have to paraphrase the introduction.

Introduction:

Here is the same information formed into a conclusion:

direct question essay example

That’s it. We’ve completed our essay. Here it is with the 4 paragraphs put together.

Finished IELTS double question essay.

direct question essay example

     (351 words)

Go through this lesson as many times as you need to in order to fully understand it and put in lots of practice writing IELTS double question essays from past exam questions. Practice is the only way to improve your skills.

Would you prefer to share this page with others by linking to it?

  • Click on the HTML link code below.
  • Copy and paste it, adding a note of your own, into your blog, a Web page, forums, a blog comment, your Facebook account, or anywhere that someone would find this page valuable.

Like this page?

More help with ielts double question essays & other task 2 essays.

IELTS Writing Task 2  – T he format, the 5 question types, the 5 step essay writing strategy & sample questions. All the key information you need to know.

The 5 Types of Task 2 Essay   – How to recognise the 5 different types of Task 2 essays. 15 sample questions to study and a simple planning structure for each essay type.

Understanding Task 2 Questions  – How to quickly and easily analyse and understand IELTS Writing Task 2 questions.

How To Plan a Task 2 Essay  – Discover why essay planning is essential & learn a simple 4 step strategy, the 4 part essay structure & 4 methods of generating ideas.

How To Write a Task 2 Introduction  – Find out why a good introduction is essential. Learn how to write one using a simple 3 part strategy & discover 4 common mistakes to avoid.

How To Write Task 2 Main Body Paragraphs  – Learn the simple 3 part structure for writing great main body paragraphs and also, 3 common mistakes to avoid. 

How To Write Task 2 Conclusions  – Learn the easy way to write the perfect conclusion for a Task 2 essay. Also discover 4 common mistakes to avoid.

Task 2 Marking Criteria  – Find out how to meet the marking criteria in Task 2. See examples of good and poor answers & learn some common mistakes to avoid.

The 5 Task 2 Essay Types:

Step-by-step instructions on how to plan & write high-level essays. Model answers & common mistakes to avoid.

   Opinion Essays

   Discussion Essays

  Problem Solution Essays

  Advantages & Disadvantages Essays

  Double Question Essays

Other Related Pages

IELTS Writing Test  – Understand the format & marking criteria, know what skills are assessed & learn the difference between the Academic & General writing tests.

  • IELTS Writing
  • Double Question Essays
  • Back To Top

 * New * Grammar For IELTS Ebooks

direct question essay example

$9.99 each       Full Set   Just   $ 23.97

Find Out More >>

IELTS Courses

direct question essay example

Full details...

direct question essay example

IELTS Writing Ebook

direct question essay example

Discount Offer

$7 each       Full Set Just   $ 21

direct question essay example

Find out more >>

Testimonials

“I am very excited to have found such fabulous and detailed content. I commend your good work.”  Jose M.

“Thanks for the amazing videos. These are ‘to the point’, short videos, beautifully explained with practical examples."  Adari J.

"Hi Jacky, I bought a listening book from you this morning. You know what? I’m 100% satisfied. It’s super helpful. If I’d had the chance to read this book 7 years ago, my job would be very different now."  Loi H.

"Hi Jacky, I recently got my IELTS results and I was pleased to discover that I got an 8.5 score. I'm firmly convinced your website and your videos played a strategic role in my preparation. I was able to improve my writing skills thanks to the effective method you provide. I also only relied on your tips regarding the reading section and I was able to get a 9! Thank you very much." Giano

“After listening to your videos, I knew I had to ditch every other IELTS tutor I'd been listening to. Your explanations are clear and easy to understand. Anyways, I took the test a few weeks ago and my result came back: Speaking 7, listening 9, Reading 8.5 and Writing 7 with an average band score of 8. Thanks, IELTS Jacky." Laide Z.

      Contact

      About Me

      Site Map

      Privacy Policy

      Disclaimer

IELTS changes lives.

Let's work together so it changes yours too.

Copyright  © 2024     IELT Jacky     

All Right Reserved

IELTS is a registered trademark of the University of Cambridge, the British Council, and IDP Education Australia. This site and its owners are not affiliated, approved or endorsed by the University of Cambridge ESOL, the British Council, and IDP Education Australia.

direct question essay example

25,000+ students realised their study abroad dream with us. Take the first step today

Meet top uk universities from the comfort of your home, here’s your new year gift, one app for all your, study abroad needs, start your journey, track your progress, grow with the community and so much more.

direct question essay example

Verification Code

An OTP has been sent to your registered mobile no. Please verify

direct question essay example

Thanks for your comment !

Our team will review it before it's shown to our readers.

Leverage Edu

  • Study Abroad Test Prep /

Direct Question Essay in IELTS: Everything You Need to Know

direct question essay example

  • Updated on  
  • Dec 30, 2023

direct question essay ielts

The IELTS Writing Task 2 demands test takers to write cohesive, nuanced, and insightful essays. Within this section, a diverse spectrum of essay types arises, such as opinion pieces, problem-solution analyses, and advantage/disadvantage examinations. Notably, amongst these stands the Direct Question Essay, characterized by its explicit and directive nature, demanding a clear and concise response within a 250-word limit. To know more about IELTS direct question essays, read the entire blog. 

This Blog Includes:

Top 5+ tips for direct question essay ielts, structure of an ielts direct question essay, scoring criteria for direct essay questions, most common ielts direct essay sample questions, ielts direct question essay samples .

Here are the top tips for direct question essays in IELTS writing task 2. Refer to them to bolster your chances of scoring better in IELTS writing. 

  • Take a good look at the test questions before diving in. It’ll save you time and frustration in the long run, and make sure you’re answering what the examiners want to know.
  • You should keep a formal tone while writing direct-question essays in IELTS writing task 2. 
  • Individuals must stick to the given word limit. Exceeding the word limit can cause you to lose marks and time. Write your essay with a word count of at least 250-270 words. 
  • Set a time restriction during your preparation to enhance your overall speed and accuracy. During your actual exam, you will get around 40 minutes to write a direct-question essay. Therefore, practising under time constraints can help you get accustomed to the actual time crunch you’ll face during the test. 
  • A proper content structure must be followed, i.e., begin with an introduction paragraph, a two-body paragraph and end the essay with a concise conclusion.  
  • Make your intro concise and to the point. In other words, keep them tight, clear, and accurate to the topic. 
  • The main body paragraphs of your essay should answer the questions asked in the question.
  • Refined writing with proper grammar and vocabulary makes a big impression and showcases your language proficiency. 

Sticking to the expected structure in IELTS direct question essays is crucial to avoid losing points and improve your chances of scoring better. Refer to the data given below to get a better understanding of the structure of IELTS Direct Question Essays. 

Introduction Paragraph

Always start your essay with a concise, articulated and punchy introduction. Intros that pack a punch can rope in a lot of eyeballs and encourage them to read further. Begin with a succinct yet insightful phrasing of the assigned prompt, then offer a taste of the key arguments you’ll elaborate upon in the subsequent paragraphs. 

Body Paragraph 1

Directly address the first question in the opening paragraph with a clear and comprehensive response. 

Body Paragraph 2

In this paragraph, feel free to explore your second or third questions, bolstering your arguments with illustrative examples for greater clarity and impact.

Conclusion Paragraph

Sum up your entire essay in this paragraph. Ensure that you have included all the major points in this part of the essay. 

The IELTS Direct Essay Questions are evaluated on the following factors: 

Task Achievement 

Your essay should answer all the questions asked on the topic. You should use linkers and examples to make your essay more cohesive, comprehensible and impactful. 

Lexical Resource

Diversify your lexical choices to enhance the persuasiveness and sophistication of your written arguments.

Grammatical Range and Accuracy

Evaluate your essay for grammatical inconsistencies that might detract from its persuasiveness and accuracy.

Maintain Coherence

Your IELTS Direct Question Essay should be coherent and comprehensible to the readers. It must be segregated into concise and articulated paragraphs that are easy to read and comprehend.

Here are the most common IELTS Direct Essay Sample Questions. 

Refer to the following samples to get a solid idea about IELTS direct question essays and how to answer them. 

Q: Online shopping is significantly increasing. What impact does this have on people, what are the disadvantages, and what are the job opportunities?

Ans: The internet’s discovery has transformed numerous facets of people’s lives, giving rise to online shopping and sparking significant societal changes. While it delivers numerous benefits, it also introduces disadvantages that warrant discussion.

One of the main disadvantages of online shopping is that it lacks a tangible experience as consumers can not touch, feel, or try the items they are purchasing beforehand. It not only causes dissatisfaction but has also increased return rates. In addition to this, the impersonal nature of online shopping has led to poor customer service. Apart from this, another significant concern of people is the risk of fraud and identity theft that it carries. With the growing prevalence of online transactions, consumers face a heightened risk of compromising their personal and financial information.

Despite these challenges, the rise of online shopping has created numerous job opportunities. The e-commerce industry is booming and requires a wide range of skills such as web development, digital marketing, logistics, and customer service. Due to this, new jobs have been created, allowing individuals to start their businesses. Furthermore, the need for improved cybersecurity has led to an increase in demand for this field. In today’s fast-paced world, businesses are emphasizing safeguarding their customers’ data and ensuring a secure online shopping experience, making cybersecurity an essential and integral part of the e-commerce industry.

In conclusion, although online shopping comes with a share of drawbacks, it also brings opportunities for various employment prospects. It is crucial for both consumers and businesses to effectively tackle these challenges while capitalizing on the opportunities provided by this expanding industry.

Q: Fast food is a part of life in many places. Some people think this has bad effects on lifestyle and diets. How is the consumption of fast food bad for people? Why should one avoid fast food? 

Ans : The omnipresence of fast food in today’s society is undeniable, as it has become an integral part of every individual’s life worldwide. However, fast food’s detrimental effects on our lifestyle and diet are unmistakable, and I wholeheartedly agree with this viewpoint for several reasons.

Fast food is often high in sugar, unhealthy fats, and calories while providing very few nutrients. If someone consumes it regularly, there is a high probability that it can lead to obesity, heart disease, diabetes, and other health-related problems. It is concerning as in the present scenario many countries are facing high rates of childhood obesity cases. In addition to this, the convenience of fast food has encouraged a sedentary lifestyle. Daily consumers tend to invest less time in preparing home-cooked meals and participating in physical fitness activities, contributing to social isolation. This lifestyle shift can have adverse effects on people’s overall well-being.

Moreover, fast food promotes overconsumption through clever marketing strategies. For instance, the ubiquitous practice of offering “value meals” with larger portions at only a slightly higher cost may entice customers to opt for larger servings than they need. Coupled with the addictive appeal of certain fast food ingredients, such as the high levels of salt, sugar, and fat, this marketing approach can significantly contribute to overeating and, consequently, weight gain.

In conclusion, while fast food may be an enticing option for many individuals, it comes with adverse impacts on health and overall lifestyle. Therefore, people need to make conscious decisions regarding their food consumption, and governments should actively play a pivotal role in promoting healthy eating habits. This may involve enacting regulations on fast food advertising, providing education, and encouraging physical activity.

With that, we conclude our foray into IELTS Direct Question Essays. 

Ans. The IELTS direct-question essay is divided into an introduction, two body paragraphs and a conclusion. 

Ans. Test takers can use a range of linkers, adverbial phrases, references, and punctuations to bolster their overall scores in IELTS writing. 

Ans. The IELTS Writing Task 2 requires individuals to complete the task within 40 minutes. 

Related Reads:

Visit the Leverage Live page of Leverage edu or contact our study abroad experts at 1800-57-2000 to strengthen your scores and application to secure your spot in your dream college. 

' src=

Shubham Das

Shubham Das has been working as an educational content writer for the past two years and has a background in filmmaking & screenplay/ teleplay writing. He is fascinated by the human psyche, literature and cinema.

Leave a Reply Cancel reply

Save my name, email, and website in this browser for the next time I comment.

Contact no. *

direct question essay example

Connect With Us

25,000+ students realised their study abroad dream with us. take the first step today..

direct question essay example

Resend OTP in

direct question essay example

Need help with?

Study abroad.

UK, Canada, US & More

IELTS, GRE, GMAT & More

Scholarship, Loans & Forex

Country Preference

New Zealand

Which English test are you planning to take?

Which academic test are you planning to take.

Not Sure yet

When are you planning to take the exam?

Already booked my exam slot

Within 2 Months

Want to learn about the test

Which Degree do you wish to pursue?

When do you want to start studying abroad.

January 2024

September 2024

What is your budget to study abroad?

direct question essay example

How would you describe this article ?

Please rate this article

We would like to hear more.

Have something on your mind?

direct question essay example

Make your study abroad dream a reality in January 2022 with

direct question essay example

India's Biggest Virtual University Fair

direct question essay example

Essex Direct Admission Day

Why attend .

direct question essay example

Don't Miss Out

TED IELTS

  • A Beginner’s Guide to IELTS
  • Common Grammar Mistakes [for IELTS Writing Candidates]

Writing Correction Service

  • Free IELTS Resources
  • Practice Speaking Test

Select Page

IELTS Writing Task 2: Two-Part Questions

Posted by David S. Wills | Jul 18, 2018 | IELTS Tips , Writing | 1

IELTS Writing Task 2: Two-Part Questions

Today we’re going to look at an IELTS writing task 2 question that often makes students a little worried: the two-part question . This is also sometimes referred to as “the direct question” as it is more direct than other types of writing questions, such as advantages and disadvantages or causes and solutions .

In this lesson, I will show you what this question looks like and explain how to answer it.

Table of Contents

What is a two-part question, two-part question examples, analysing the question, structuring a two-part question essay, sample answer.

In IELTS writing task 2, there are different sorts of questions you may be asked. One of them is called the two-part question (or sometimes “the direct question”). It is most commonly called a two-part question because it contains two distinct questions.

Of course, some other questions also contain two parts. For example, a problem and solution essay is two parts. However, what we mean by a “two-part question” is one that contains two questions . The reason this is sometimes called a “direct question” task is that the questions themselves are more direct than other IELTS writing task 2 question types, which instruct candidates to explore an idea. This is rather vague , whereas the two-part question is very specific .

To understand this idea better, let’s look at an example two-part question:

In education and employment, some people work harder than others. Why do some people work harder? Is it always a good thing to work hard?

Ok, the first thing that you may notice is that there are three parts to this question! However, the first part is actually a lead-in statement. It is not particularly important. The parts that you need to address in your essay are the two questions. This is why some people say “two-part question” and others say “direct question”.

Here is another example:

Happiness is often considered difficult to define. Why is this? What factors determine happiness?

Here the first question contains a pronoun, “this”, which refers back to the idea in the first sentence. In other words, the first question is “Why is it often considered difficult to define happiness?”

Finally, here is one more example question:

Success is often measured by wealth and material possessions. Do you think wealth is the best measure of success? What makes a successful person?

As you can see by now, each question has three parts: an introductory statement and two individual (but related) questions. You will see this exact format used with numerous IELTS topics .

Sometimes the lead-in statement will be very long, and sometimes the questions are quite closely linked. Other times the statement may be short or the questions ask quite different things.

How to Answer Two-Part Questions

The first thing to remember is: DON’T PANIC! Two-part questions often make students a bit nervous, but really they are not so bad. They are no more difficult to answer than the other IELTS task 2 question types.

In fact, this type of question may even be the easiest one to answer!

Although the questions themselves are sometimes a bit challenging, they are direct questions . This means that it is less likely you will stray off-topic while writing. In other words, your task is actually more straightforward than it would be with an agree/disagree question, for example.

Let’s look at an example question so we can analyse it. This is the first thing you should do in any IELTS writing task 2 essay, and it’s important to spend a minute or two thinking carefully about it.

Some parents buy their children whatever they ask for, and allow their children to do whatever they want. Is this a good way to raise children? What consequences could this style of parenting have for children as they get older?

We can take three steps to analyse this question.

  • Let’s look at the first sentence: What is it about? It is about permissive parents – ie parents who give their children too much (or who give in to their child’s demands) and let them do too many things.
  • Next, the first question contains a pronoun (“this”) which refers to the ideas in the first sentence, and asks if it’s good or not.
  • The second question talks about the consequences – not for the parents, but for the children.

I have deliberately chosen a slightly difficult question here in order to show you how to answer it. Usually, the questions are a little more direct and obvious, but here you need to consider the three sentences in order, making sure to understand each fully.

In a situation like this, if you make an incorrect assumption about the first or second parts of the sentence, it may cause a serious lack of coherence and cohesion , in addition to poor task achievement .

For example, if you thought it was just about children not being made to study enough, you might write the next paragraph all about a lack of studying and the following one may come back closer to the correct idea because of the more specific question. This would look bad, indicating a lack of unity in your writing.

Thankfully, it is not difficult to create a perfect two-part essay structure. In fact, it is very easy!

Here is a basic essay template:

In a previous article, I talked about whether to use a 4 or 5 paragraph essay . This is one case when you definitely want to use a four-paragraph essay.

Here’s the video, if you want to review it:

I have lots more posts on the topic of IELTS essay structures. Here are a few: 

  • IELTS Writing Task 2 Essay Structures
  • Planning IELTS Writing Task 2 Structure

How to Structure a Paragraph

In this section, I will give you my sample answer to the above question, using the basic four-paragraph structure that I mentioned. Obviously, my ideas will be different to yours, but the structure which I outlined can be used 100% of the time.

In today’s society, some parents are becoming increasingly permissive. They do not impose sufficient discipline on their children, and in some cases buy them too many things. This essay will explore why this is not a good way to raise children and why it will have negative impacts upon them in future. While it is understandable that parents want to give their children expensive toys and let them run freely in the streets, this is actually not really an appropriate method of parenting. Having too many toys encourages children to be materialistic and does not offer them the same change to develop social skills like sharing. Furthermore, when children have asked for the toys, it gives them a sense of entitlement and even power over their parents. As for giving children too much freedom, there are obviously a great many dangers in this world from which they need to be kept safe. Children also need rules and boundaries to encourage them to develop into mature and responsible adults. If parents insist on this permissive style of parenting, their children will grow up with very different values from those of stricter parents. Children who were never forced to study will end up with poorer grades in school, and those who were given everything they wanted as children will expect everything to come easily as adults. In short, they will lead difficult and disappointing lives, in contrast with what their parents hoped. In conclusion, although it is tempting to give children whatever they want, parents ought to set rules and boundaries, and to be careful with how they reward children. If parents fail to impose a basic level of discipline, children may grow up with a poor attitude that will cause them and others to suffer.

There are really no words or phrases that are unique to the two-part question essay. Just use regular academic English as with any other IELTS task 2 question, and of course stick to the topic.

For my essay, I used the word “permissive.” It means allowing too much and not setting enough rules. You could use similar words like “liberal,” “easygoing”, “live and let live”, and so on, although they vary slightly in precise meaning.

Here are some more useful terms that you can use to talk about parents:

direct question essay example

As always, remember to avoid IELTS phrases and other cliches. Learn new vocabulary by topic and in collocations rather than isolation.

Tips for Two-Part Questions

Finally, a few tips to remember:

  • Practice this question type often before the exam – it’s pretty common!
  • Make sure you understand each part of the question before answering.
  • Always use the four-paragraph essay structure.
  • Work on idea generation at home in order to come up with good, on-topic answers.

I made this article into a short video. Please give it a ‘LIKE’ and subscribe to the channel if you find it useful.

About The Author

David S. Wills

David S. Wills

David S. Wills is the author of Scientologist! William S. Burroughs and the 'Weird Cult' and the founder/editor of Beatdom literary journal. He lives and works in rural Cambodia and loves to travel. He has worked as an IELTS tutor since 2010, has completed both TEFL and CELTA courses, and has a certificate from Cambridge for Teaching Writing. David has worked in many different countries, and for several years designed a writing course for the University of Worcester. In 2018, he wrote the popular IELTS handbook, Grammar for IELTS Writing and he has since written two other books about IELTS. His other IELTS website is called IELTS Teaching.

Related Posts

How to Structure a Paragraph

November 12, 2016

STOP Using Vague Language for IELTS Writing Task 1

STOP Using Vague Language for IELTS Writing Task 1

June 20, 2022

IELTS Topics: Pets

IELTS Topics: Pets

June 20, 2020

Describe your Hometown [IELTS Cue Card]

Describe your Hometown [IELTS Cue Card]

February 4, 2020

Mariam

could someone evaluate this and give the band score it deserves? here is my writing of the given topic….. In education and employment, some people work harder than others. 1. Why do some people work harder? 2. Is it always a good thing to work hard?

ans : It is often seen that some people put in more effort than others in the field of education and employment . The possible reasons could be due to the workload pressure or genuine passion .While passion for accomplishing task is a good thing, working hard due to constant pressure and other circumstances could be a problem . This essay explores the potential reasons as to why some work more than others and also discusses whether it is good to do so or not.

While students work hard due to their caliber and interest in studies working people work hard due to circumstances .Although there are a few students who study due to constant nagging by mentors or parents, most of them develop a genuine interest towards the subjects. Employers, on the other hand, work up to their innate potential to make both ends meet. For example, some workers work overtime that is for longer hours in order to earn higher salary or during the weekends to cope with the pending works . Sometimes they complete their tasks in advance in case they have any upcoming projects or plans outside the workplace to compensate for the missing working days.

Whether it is a good thing or a bad thing completely depends on the situation. Any work, no matter how important and urgent, if done against will and under high pressure will never give good outcomes . Instead it will lead to anxiety and stress if not accomplished on time. But if one is passionate and hungry for success and enjoys pursuing their interest, it will undoubtedly bear fruits and their hard work will eventually pay off.

All things considered, hard work is the key to success with respect to both education and employment. But it is important to make a note that one should have a great deal of passion and sincerity to achieve success and should not be forced to work due to helplessness or under pressure.

Leave a reply Cancel reply

Your email address will not be published. Required fields are marked *

This site uses Akismet to reduce spam. Learn how your comment data is processed .

Download my IELTS Books

books about ielts writing

Recent Posts

  • How to Improve your IELTS Writing Score
  • Past Simple vs Past Perfect
  • Complex Sentences
  • How to Score Band 9 [Video Lesson]
  • Taxing Fast Food: Model IELTS Essay

ielts writing correction service

Recent Comments

  • Francisca on Adverb Clauses: A Comprehensive Guide
  • Mariam on IELTS Writing Task 2: Two-Part Questions
  • abdelhadi skini on Subordinating Conjunction vs Conjunctive Adverb
  • David S. Wills on How to Describe Tables for IELTS Writing Task 1
  • anonymous on How to Describe Tables for IELTS Writing Task 1
  • Lesson Plans
  • Model Essays
  • TED Video Lessons
  • Weekly Roundup

IELTS Exams logo

Direct Question Essay IELTS 2024: Tips, Structure, Sample

Looking for Direct Question Essay IELTS tips Click to learn how to plan, recognise prompts get samples for a successful IELTS essay!

9/26/2023 2 min read

direct question essay example

In the world of standardized English language proficiency tests, the IELTS (International English Language Testing System) is a formidable challenge for many. One of its key components, the Direct Question Essay, is often a source of anxiety for test-takers. But fear not! In this comprehensive guide, we will unravel the intricacies of the Direct Question Essay in IELTS, offering valuable tips and insight into its structure and providing sample questions with model answers.

Understanding the Direct Question Essay

What is a direct question essay in ielts.

The Direct Question Essay, or the Opinion Essay, is an essay task in the IELTS writing section. It presents a clear and straightforward question that requires a direct response. Test-takers are expected to provide their opinion, supported by relevant arguments and examples.

Why is it Important?

The Direct Question Essay assesses a candidate's ability to express opinions coherently, develop arguments logically, and use appropriate vocabulary and grammar. It's crucial to perform well in this task to achieve a high band score in the IELTS writing module.

Tips for Success

Analyze the Question Carefully

Before diving into your response, take a moment to understand the question thoroughly. Identify keywords and phrases that define the scope of your answer.

Plan Your Response

Outline your essay with a clear introduction, body paragraphs, and a conclusion. This structure helps organize your thoughts and ensures a coherent essay.

Provide a Clear Opinion

In the introduction, state your opinion on the given topic. Make it concise and specific, setting the tone for the rest of the essay.

Support Your Opinion

Back up your opinion with reasons and examples. Use real-life experiences or hypothetical situations to illustrate your points.

Use Formal Language

While maintaining a conversational tone, remember that IELTS essays require formal language. Avoid slang and contractions.

Vary Sentence Structure

Enhance the readability of your essay by using a variety of sentence structures. Combine simple, compound, and complex sentences for better flow.

Proofread and Edit

Allocate time for proofreading. Check for spelling, grammar, and punctuation errors. Ensure your essay is polished and error-free.

Structure of a Direct Question Essay

The structure of a Direct Question Essay typically consists of:

Introduction

Introduce the topic and provide a clear opinion.

Body Paragraphs

Each paragraph should focus on a single supporting point.

Provide examples and evidence to strengthen your arguments.

Summarize your main points and restate your opinion.

Sample Questions and Model Answers

Question 1:.

"Do you agree or disagree with the statement that technology has made people more isolated?"

Model Answer:

Technology has made people more isolated. In today's digital age, individuals are often engrossed in their smartphones and computers, limiting face-to-face interactions. For instance, while connecting people virtually, social media platforms have also contributed to a decline in genuine personal connections.

Question 2:

"Is it better to study alone or in a group?"

Studying in a group yields better results. When you study with others, you can share ideas, clarify doubts, and gain different perspectives. Moreover, group study sessions encourage discipline and time management.

Mastering the Direct Question Essay in IELTS requires practice and a clear understanding of its structure and requirements. By following these tips and reviewing sample questions and model answers, you can boost your confidence and perform exceptionally well in this challenging task.

Subscribe newsletter

IELTS Preparation with Liz: Free IELTS Tips and Lessons, 2024

' src=

  • Test Information FAQ
  • Band Scores
  • IELTS Candidate Success Tips
  • Computer IELTS: Pros & Cons
  • How to Prepare
  • Useful Links & Resources
  • Recommended Books
  • Writing Task 1
  • Writing Task 2
  • Speaking Part 1 Topics
  • Speaking Part 2 Topics
  • Speaking Part 3 Topics
  • 100 Essay Questions
  • On The Day Tips
  • Top Results
  • Advanced IELTS

100 IELTS Essay Questions

Below are practice IELTS essay questions and topics for writing task 2. The 100 essay questions have been used many times over the years. The questions are organised under common topics and essay types. IELTS often use the similar topics for their essays but change the wording of the essay question.

In order to prepare well for writing task 2, you should prepare ideas for common topics and then practise applying them to the tasks given (to the essay questions). Also see model essays and tips  for writing task 2.

Below you will find:

  • Essay Questions By Topic
  • Essay Questions by Essay Type

Please also note that my new Grammar E-book is now available in my store along with my Ideas for Essay Topics E-book and Advanced Writing Lessons. To visit store, click here: Liz’s Store

1) Common IELTS Essay Questions

IELTS practice essay questions divided by topic. These topics have been reported by IELTS students in their tests. Essay questions have been recreated as accurately as possible.

  • Art   (5 essay questions)
  • Business & Money   (17 essay questions)
  • Communication & Personality   (20 essay questions)
  • Crime & Punishment   (12 essay questions)
  • Education   (17 essay questions)
  • Environment   (12 essay questions)
  • Family & Children   (8 essay questions)
  • Food & Diet (13 essay questions)
  • Government (6 essay questions)
  • Health   (9 essay questions)
  • Housing, Buildings & Urban Planning (8 essay questions)
  • Language (6 essay questions)
  • Leisure (1 essay question)
  • Media & Advertising   (12 essay questions)
  • Reading  (5 essay questions)
  • Society   (10 essay questions)
  • Space Exploration (3 questions)
  • Sport & Exercise   (6 essay questions)
  • Technology  (6 essay questions)
  • Tourism and Travel   (11 essay questions)
  • Transport  (7 essay questions)
  • Work (17 essay questions)

2) IELTS Essay Questions by Essay Type 

There are 5 main types of essay questions in IELTS writing task 2 (opinion essays, discussion essay, advantage/disadvantage essays, solution essay and direct question essays). Click on the links below to see some sample essay questions for each type.

  • Opinion Essay Questions
  • Discussion Essay Questions
  • Solution Essay Questions
  • Direct Questions Essay Titles 
  • Advantage / Disadvantage Essay Questions

………………………………

FREE SUBSCRIBE : Get New Lessons & Posts by Email

Type your email…

Advanced IELTS Lessons & E-books

direct question essay example

Click Below to Learn:

  • IELTS Test Information

Copyright Notice

Copyright © Elizabeth Ferguson, 2014 – 2024

All rights reserved.

Privacy Policy & Disclaimer

  • Click here:  Privacy Policy 
  • Click here: Disclaimer

Return to top of page

Copyright © 2024 · Prose on Genesis Framework · WordPress · Log in

IELTS Direct Essay on Business With Sample Answer

In a Direct Question essay, two or three direct questions are based on a given statement. Find ways to answer such a question and improve band.

Share this article:

About the article

Direct question essay example, question: .

Companies use a variety of methods to improve the sales of their products.

What are those methods? Which is the most effective method?

Many businesses choose an array of techniques to enhance the sales of their merchandise. Advertisements and social media strategies are a few ways used by different organisations to boost up the sales and in my opinion showcasing the product online turns out to be the most fruitful method.

Many companies rely on advertisements, TV, social media, google adwords and billboards in order to popularise their products. One of the most popular methods – TV commercials often feature famous actors, sports stars and other well-known personalities, to try and persuade people to buy products, as fans of these people are more likely to buy something that they are endorsing. For instance, a report by a nutrition brand revealed that their sales of healthy snack products reached an all-time high when they chose Indian cricketer Virat Kohli as their product endorser.

While the most effective modern method of advertising is online. Companies spend billions on Google Adwords and marketing through social media. The companies that use this method are pushing their products in front of a huge audience, as social media apps are more widely used and downloaded than any TV station. They also have the chance of the product post going viral and being shared millions of times, which then promotes itself. For example, these days Facebook and Instagram have been effective tools in launching new products to the age group 18-25 since most young people are active on social media.

In conclusion, there are varieties of practices that can make a product popular with its consumers. The use of social media is the most effective tool in advertising in 2021 for it can reach all people in any part of the world.

  • Practice Test
  • Useful Tips – Tricks
  • Full Writing Review
  • General Writing Task
  • Writing Task 1
  • Writing Task 2
  • Writing Exercises
  • Writing Sample – Topics
  • Writing Vocabulary
  • Speaking Vocabulary
  • Intro Question
  • Speaking Part 1
  • Speaking Part 2
  • Speaking Part 2 – Audio
  • Speaking Part 3
  • IELTS Books
  • Recent Exams
  • IELTS Vocabulary
  • Essay from Examiners
  • IELTS Ideas

Logo

IELTS App - For Mobile

Ready for the IELTS exam with our IELTS app. Over 2 million downloads

Download App

Popular Last 24h

In many countries,today there are many highly qualified graduates without employment., describe something difficult you would like to succeed in doing, describe a person whom you met for the first time and made you happy, writing task 2: film stars and celebrities often share their views on public matters that have little to do with their profession., describe what you think would be the perfect job (or “dream job”) for you., makkar ielts speaking jan to apr 2024 final version [pdf], first mobile phone.

  • IELTS Test/Skills FAQs
  • IELTS Scoring in Detail
  • Forecast Speaking – 2023
  • List IELTS Speaking Part 3
  • List IELTS Speaking Part 1
  • IELTS Writing 2023 – Actual Test

Our Telegram

Join our community for IELTS preparation and share and download materials.

The information on this site is for informational purposes only. IELTS is a registered trademark of the University of Cambridge ESOL, the British Council, and IDP Education Australia. This site and its owners are not affiliated, approved or endorsed by University of Cambridge ESOL, the British Council, or IDP Education Australia.

Latest Articles

Writing task 1: average number of hours students, writing task 1: internet use for different purposes in australia, ielts speaking part 3: topic teamwork, most popular, describe a film that made you laugh, topic: experience is the best teacher.

ieltspracticeonline All Rights Reserved

direct question essay example

IELTS Direct Question Essay Model Answer: Economics

In this post, we will look at a direct question essay example from the IELTS writing task 2 test. Students often ask if the questions are repeated year after year and the answer is no, but the topics are. There are so many questions written each year, you may find your practice answering various questions on different topics. For example, you could write essays to answer questions about education or the environment, which benefits you because you learn vocabulary associated with those topics and develop ideas that can help you in your writing test.

Practising writing IELTS task 2 essays on a range of topics is a great way to learn new vocabulary for those topics, but also to practice your essay structures. You begin to develop your ideas around those topics, thinking of examples and giving your opinions.

If you would like to purchase a 29  page PDF download that is easy to read and print out please take a look at the bookshop >

If you would like to learn how to structure a direct question essay go to the resource here: How to structure a direct question essay

Take a look at the IELTS Direct Question Essay example below:

Wealth and material belongings often measure the success that a person has. Do you think wealth is the best measure of success? What makes a successful person?

Take a look at the model answer.

Riches and possessions are frequently viewed as the gauge for the achievements of an individual. Firstly, this essay will discuss how having a lot of money can be a benchmark for achievement and secondly, discuss how a person can be seen as accomplished. On the one hand, when a person has accrued a vast amount of wealth, this is often shown through their belongings or property. Owning a large property or many properties, the latest cars and other material possessions can show the world that you are successful. Being able to show your achievements through your good fortune, can propel you into a different class and do things that not everyone is able to do. For example, Virgin CEO Richard Branson’s success was documented by Forbes magazine in 2017, as he was able to purchase more than one private island in the Caribbean that year. In my opinion, I believe that success through business can easily be gauged by the amount of wealth one has. On the other hand, in my view, accomplishments can be also seen in different forms. It is true that money can gauge success, although there are also other factors to consider. Successful people are generally positive, motivated, enthusiastic and problem solvers. They take risks because they know that to further themselves, reaching goals and realising aspirations takes hard work, determination and not being scared of failure. For instance, The Economist in 2017 reported that 80% of successful business leaders had positive outlooks and saw themselves as risk-takers.  In conclusion, it is a fact that having a large amount of money can measure one’s achievements, however, in order to get to that stage in life, people also need to realise the determining factors that make a person become a success. 

(Word count – 293 / Band score 8)

The following video will give you contains sample Essay in audiovisual format.

Instructor Feedback on IELTS Direct Question Essay: Economics

Task Achievement  – The essay provides an answer to the question asked, supported by relevant examples. Coherence and Cohesion  – The answer has been divided into clear logical paragraphs and each main body paragraph only has one main idea. There are cohesive links between the main body paragraphs . Lexical Resource  – There is evidence of a wide range of vocabulary, with no errors in the text. Grammatical Range and Accuracy  – The answer has no grammatical errors. The sentences have a wide range of structures. 

Want to check more Direct Question Essay Samples? Check the IELTS Direct Question Essay Examples

We hope you found this post useful in helping you to study for the IELTS Test . If you have any questions please let us know in the comments below or on the Facebook page.

The best way to keep up to date with posts like this is to like us on Facebook , then follow us on Instagram and Pinterest . If you need help preparing for the IELTS Test, join the IELTS Achieve Academy and see how we can assist you to achieve your desired band score. We offer an essay correction service, mock exams and online courses.

Related Posts

How To Write An Agree Or Disagree Essay

How To Write An Agree Or Disagree Essay?

In writing task 2 you will be asked to write a discursive essay (250 words…

IELTS Agree/Disagree Essay Sample 8 - Government

IELTS Cause/Effect Essay Sample 1 – Crime

IELTS Writing Task 2 cause/effect essay example that is a band score 8. The question…

Leave a Comment Cancel Reply

Your email address will not be published. Required fields are marked *

Save my name, email, and website in this browser for the next time I comment.

IMAGES

  1. How To Write A Direct Question Essay

    direct question essay example

  2. How To Write A Direct Question Essay

    direct question essay example

  3. IELTS Writing Task 2

    direct question essay example

  4. IELTS Writing Task 2 : Direct Question Essays

    direct question essay example

  5. How To Write A Direct Question Essay

    direct question essay example

  6. IELTS Direct Question Essay Sample 1

    direct question essay example

VIDEO

  1. DIRECT AND INDIRECT SPEECH PARAGRAPH EXAMPLES--1

  2. Direct essay: plan and essay

  3. Plus Two Public Exam

  4. IELTS Essay Writing Direct Question

  5. IELTS Direct Question Essay Sample: Society

  6. Direct speech Indirect speech introduction || JSJ JESY ENGLISH GRAMMAR

COMMENTS

  1. IELTS Direct Question Essays

    Direct Question Essay: A Skill to Learn! In conclusion, mastering the art of writing Direct Question Essays is achievable with practice, a clear understanding of the format, and attention to key pointers. By following the structure, incorporating the tips provided, and analyzing the sample questions and answers, you can significantly enhance ...

  2. IELTS Direct Questions Sample Essay Titles

    Below is a list of sample essay questions for IELTS direct questions essays which can come in writing task 2. These essay types are also known as two question essays (although it is possible to get more than two questions). Direct Questions Essay Tips. spend time planning the answers to the questions; each question must have one main point as ...

  3. IELTS Direct Question Essay: tips, common mistakes, questions & essays

    Step 4: Structure Your Essay. The final step in the planning process is to structure your essay. This simply means deciding which main ideas to put in which paragraphs. I would recommend a simple structure like this: Paragraph 1: introduce the essay. Paragraph 2: discuss your answer to the 1st question.

  4. Direct Question Essay IELTS 2022: Tips, Structure, Sample Questions

    Direct Questions are the specific two-part questions asked in IELTS Writing Exam Part 2. 40 minutes are provided to write an essay of 250 words for Direct Question IELTS. You can practice for the Direct Question Essay IELTS by writing on various topics available, including the internet itself, lifestyle, success, news, happiness, industry ...

  5. IELTS Double Question Essays

    IELTS double question essays are also known as 'direct question' or 'two questions' essays. They are distinguished by two characteristics: ... The 5 Types of Task 2 Essay - How to recognise the 5 different types of Task 2 essays. 15 sample questions to study and a simple planning structure for each essay type.

  6. Direct Question Essay in IELTS: Everything You Need to Know

    You should keep a formal tone while writing direct-question essays in IELTS writing task 2. Individuals must stick to the given word limit. Exceeding the word limit can cause you to lose marks and time. Write your essay with a word count of at least 250-270 words. Set a time restriction during your preparation to enhance your overall speed and ...

  7. How To Write A Direct Question Essay

    You must read the question and answer the questions asked in your essay. Take 5 minutes and plan out your ideas and examples. TIP >> It is very important that spend a full 40 minutes on this task as the score you get for writing task 2 is two-thirds of your total writing score. You also need to write a minimum of 250 words and use your own ...

  8. IELTS Model Essay Score 9 for Direct Questions

    For example "Discuss both sides and give your opinion" is categorised by many teachers as a Discussion Essay, but it still requires you to give an opinion. You were given a "Two Question Essay" or a "Direct Questions Essay". Please remember that IELTS do not categorised essays - teachers do.

  9. IELTS Writing Task 2: Two-Part Questions

    For example, a problem and solution essay is two parts. However, what we mean by a "two-part question" is one that contains two questions. The reason this is sometimes called a "direct question" task is that the questions themselves are more direct than other IELTS writing task 2 question types, which instruct candidates to explore an idea.

  10. Direct Question Essay Examples

    IELTS Direct Question Essay Sample 4 - Education. IELTS Writing Task 2 direct question essay example that is a band score 8. The question is: Some people think that children should be homeschooled when they are very young while others think it is better for them to attend a kindergarten. Which do you think is better?

  11. Direct Question Essay IELTS 2024: Tips, Structure, Sample

    Structure of a Direct Question Essay. The structure of a Direct Question Essay typically consists of: Introduction. Introduce the topic and provide a clear opinion. Body Paragraphs. Each paragraph should focus on a single supporting point. Provide examples and evidence to strengthen your arguments. Conclusion.

  12. IELTS Direct Question Essay Templates for Writing Task 2 with Band 9

    Select Essay Type, sample question and fill the blocks, in the end you will see the combined essay. Why essay structure this important? IELTS essay structure is evaluated as part of the Coherence and Cohesion rubric, which contributes 25% to the overall band. A helpful IELTS Direct Question Essay Template with a free online tool to structure ...

  13. IELTS Direct Question Essay on Society With Sample Answer

    Tips to write IELTS Direct question essay: IELTS Direct Question Essay Types are also known as two question essays. Must spend time planning the answers to the questions; Put the answer to each question in a separate body paragraph; Question: In many countries, the tradition of families having meals together is disappearing. Why is this happening?

  14. IELTS Direct Question Essay Sample 5

    Instructor Feedback on IELTS Direct Question Essay: Society. Task Achievement - The essay provides an answer to the question asked, supported by relevant examples. Coherence and Cohesion - The answer has been divided into clear logical paragraphs and each main body paragraph only has one main idea. There are cohesive links between the main ...

  15. 100 IELTS Essay Questions

    There are 5 main types of essay questions in IELTS writing task 2 (opinion essays, discussion essay, advantage/disadvantage essays, solution essay and direct question essays). Click on the links below to see some sample essay questions for each type.

  16. Communication: IELTS Direct Question Essay Model Answer

    Instructor Feedback on IELTS Direct Question Essay: Communication. Task Achievement - The essay provides an answer to the question asked, supported by relevant examples.. Coherence and Cohesion - The answer has been divided into clear logical paragraphs and each main body paragraph only has one main idea.There are cohesive links between the main body paragraphs.

  17. IELTS Direct Question Essay on Society with Sample Answer

    Sample Answer. In recent times, there has been an emerging culture by the enthusiastic youth who are energetically drawn towards exploring the world. They often wish to travel and in order to support that they volunteer in small communities in developing nations. This is a positive movement as it is advantageous not only to the society but also ...

  18. IELTS Direct Essay on Business With Sample Answer

    Direct question essay example Question: Companies use a variety of methods to improve the sales of their products. What are those methods? Which is the most effective method? Answer: Many businesses choose an array of techniques to enhance the sales of their merchandise. Advertisements and social media strategies are a few ways used by ...

  19. IELTS Writing Task 2: Spending ( Direct questions or Two-question Essay

    Sample Answer: These days, it is common to see young people squander their time and money on keeping up with the latest trends, including buying flashy clothing and tech gadgets. From my perspective, the desire for self-expression is the root cause, and this trend brings a number of negative consequences. Due to the development of society ...

  20. Business: IELTS Direct Question Essay Model Answer

    Instructor Feedback on IELTS Direct Question Essay: Business. Task Achievement - The essay provides an answer to the question asked, supported by relevant examples. Coherence and Cohesion - The answer has been divided into clear logical paragraphs and each main body paragraph only has one main idea. There are cohesive links between the main ...

  21. IELTS Task 2 Question Types: Two Questions (Double Questions)

    by Dave | Understanding Task 2 Writing | 2 Comments. I have listed here IELTS task 2 questions that contain two questions (double questions) from the real IELTS exam over the last few years - have fun learning about a common IELTS task type! Enjoy and consider signing up for my Patreon Ebooks here. Dave.

  22. IELTS Direct Question Essay Model Answer: Education and Teaching

    Instructor Feedback on IELTS Direct Question Essay: Education and Teaching. Task Achievement - The essay provides an answer to the question asked, supported by relevant examples.. Coherence and Cohesion - The answer has been divided into clear logical paragraphs and each main body paragraph only has one main idea.There are cohesive links between the main body paragraphs.

  23. IELTS Direct Question Essay Model Answer: Economics

    Instructor Feedback on IELTS Direct Question Essay: Economics. Task Achievement - The essay provides an answer to the question asked, supported by relevant examples. Coherence and Cohesion - The answer has been divided into clear logical paragraphs and each main body paragraph only has one main idea. There are cohesive links between the ...